7
$\begingroup$

Recently, I found the following three (conjectural) identities for $\pi^2$: $$\sum_{k=1}^\infty\frac{145k^2-104k+18}{k^3(2k-1)\binom{2k}k\binom{3k}k^2}=\frac{\pi^2}3,\tag{1}$$ $$\sum_{k=1}^\infty\frac{(42k^2-23k+3)16^k}{k^3(2k-1)\binom{2k}k\binom{4k}{2k}^2}=\frac{\pi^2}2,\tag{2}$$ $$\sum_{k=1}^\infty\frac{(92k^2-61k+9)64^k}{k^3(2k-1)\binom{2k}k\binom{3k}k\binom{4k}{2k}}=8\pi^2.\tag{3}$$ The converging rates of the series in $(1)$-$(3)$ are $4/729$, $1/64$ and $4/27$, respectively. One can easily check the identities $(1)$-$(3)$ numerically.

Let $P(k)=145k^2-104k+18$. Motivated by $(1)$, I also conjecture the following identities involving harmonic numbers:

$$\sum_{k=1}^\infty\frac{6P(k)(H_{3k-1}-H_{k-1})-232k+89}{k^3(2k-1)\binom{2k}k\binom{3k}k^2}=18\zeta(3),\tag{4}$$ $$\sum_{k=1}^\infty\frac{P(k)(H_{2k-1}-H_{k-1})-\frac{3(58k^2-40k+7)}{2(2k-1)}}{k^3(2k-1)\binom{2k}k\binom{3k}k^2}=\zeta(3),\tag{5}$$ $$\sum_{k=1}^\infty\frac{P(k)(H_{3k-1}^{(2)}-2H_{k-1}^{(2)})-\frac{17k+32}{9k}}{k^3(2k-1)\binom{2k}k\binom{3k}k^2}=\frac{7\pi^4}{180},\tag{6}$$ $$\sum_{k=1}^\infty\frac{P(k)(297H_{3k-1}^{(2)}-192H_{2k-1}^{(2)}-978H_{k-1}^{(2)})+\frac{27(180k^2+12k-35)}{(2k-1)^2}}{k^3(2k-1)\binom{2k}k\binom{3k}k^2}=\frac{167}{20}\pi^4,\tag{7}$$ and $$\sum_{k=1}^\infty\frac{P(k)H_{k-1}^{(3)}+\frac{28(2k-1)}{17k^2}}{k^3(2k-1)\binom{2k}k\binom{3k}k^2}=\frac{528\zeta(5)-46\pi^2\zeta(3)}{17},\tag{8}$$ where $H_n:=\sum_{0<j\le n}\frac1j$ and $H_n^{(m)}:=\sum_{0<j\le n}\frac1{j^m}$ for $m=2,3,\ldots$.

Let $Q(k)=(3k-1)(14k-3)=42k^2-23k+3$. Motivated by $(2)$, I conjecture the following identities: $$\sum_{k=1}^\infty\frac{16^k\left(Q(k)(H_{2k-1}-H_{k-1})-\frac{196k^2-100k+13}{6(2k-1)}\right)}{k^3(2k-1)\binom{2k}k\binom{4k}{2k}^2}=\frac{2\pi^2\log2-7\zeta(3)}6,\tag{9}$$ $$\sum_{k=1}^\infty\frac{16^k\left(Q(k)(H_{4k-1}-H_{2k-1})-\frac{28k^2-76k+19}{12(2k-1)}\right)}{k^3(2k-1)\binom{2k}k\binom{4k}{2k}^2}=\frac{2\pi^2\log2+35\zeta(3)}{12},\tag{10}$$ $$\sum_{k=1}^\infty\frac{16^k\left(Q(k)(4H_{4k-1}^{(2)}-H_{2k-1}^{(2)}-2H_{k-1}^{(2)})-\frac{6k+1}{k}\right)}{k^3(2k-1)\binom{2k}k\binom{4k}{2k}^2}=\frac{5\pi^4}{24},\tag{11}$$ $$\sum_{k=1}^\infty\frac{16^k\left(Q(k)(4H_{4k-1}^{(2)}-5H_{2k-1}^{(2)}-3H_{k-1}^{(2)})+\frac{32k(3k-1)}{(2k-1)^2}\right)}{k^3(2k-1)\binom{2k}k\binom{4k}{2k}^2}=\frac{\pi^4}{6}.\tag{12}$$

Let $R(k)=92k^2-61k+9$. Inspired by $(3)$, I also conjecture the following identities: $$\sum_{k=1}^\infty\frac{64^k\left(R(k)(3H_{3k-1}-5H_{k-1})-60k+21\right)}{k^3(2k-1)\binom{2k}k\binom{3k}k\binom{4k}{2k}}=48\pi^2\log2-56\zeta(3),\tag{13}$$ $$\sum_{k=1}^\infty\frac{64^k\left(R(k)(3H_{2k-1}-2H_{k-1})-\frac{124k^2-79k+13}{2k-1}\right)}{k^3(2k-1)\binom{2k}k\binom{3k}k\binom{4k}{2k}}=112\zeta(3),\tag{14}$$ and $$\sum_{k=1}^\infty\frac{64^k\left(R(k)(6H_{4k-1}-H_{k-1})-\frac{344k^2-281k+59}{2k-1}\right)}{k^3(2k-1)\binom{2k}k\binom{3k}k\binom{4k}{2k}}=560\zeta(3).\tag{15}$$

QUESTION. Can one prove the new identities $(1)-(15)$ via known methods (including the WZ method and hypergeometric series identies)?

Your comments are welcome!

$\endgroup$
1
  • $\begingroup$ Those series are amazing. nice work $\endgroup$ Commented Feb 26 at 23:33

1 Answer 1

10
$\begingroup$

All identities except possibly $(8)$ can be proved elegantly using Wilf-Zeilberger pairs.

To be precise, if $F(n,k), G(n,k)$ are two $\mathbb{C}$-valued functions satisfying $$\tag{*}F(n+1,k) - F(n,k) = G(n,k+1)-G(n,k)$$ then via some telescoping, one has, if $\lim_{n\to \infty} G(n,k) = 0$ for each $k\geq 0$, then $$\sum_{k\geq 0} F(0,k) = \lim_{n\to\infty} \sum_{k\geq 0} F(n,k) + \sum_{n\geq 0} G(0,n)$$


If one takes $$F(n,k) = \frac{\Gamma (-a+c+n+1) \Gamma \left(a+k+n+\frac{1}{2}\right) \Gamma (-b+c+n+1) \Gamma \left(b+k+n+\frac{1}{2}\right)}{\Gamma (a-n) \Gamma (b-n) \Gamma \left(k+2 n+\frac{3}{2}\right) \Gamma \left(c+k+2 n+\frac{3}{2}\right) \Gamma (-a-b+c+2 n+1)}$$ then one finds its WZ-mate $G(n,k)$ (here $\lim_{n\to\infty}\sum_{k\geq 0}F(n,k) = 0)$, we arrive at $$\sum_{k\geq 0}\frac{32 \left(a+d+\frac{1}{2}\right)_k \left(b+d+\frac{1}{2}\right)_k}{(2 d+2 k+1) (2 c+2 d+2 k+1) \left(d+\frac{1}{2}\right)_k \left(c+d+\frac{1}{2}\right)_k} \\ = \sum_{n\geq 1}\frac{(1-a)_{n-1} (1-b)_{n-1} (-a+c+1)_{n-1} \left(a+d+\frac{1}{2}\right)_{n-1} (-b+c+1)_{n-1} \left(b+d+\frac{1}{2}\right)_{n-1} \times P}{\left(d+\frac{1}{2}\right)_{2 n} \left(c+d+\frac{1}{2}\right)_{2 n} (-a-b+c+1)_{2 n}}$$

here $P$ is a long polynomial that pops up in computing WZ-mate $G(n,k)$: $8 a^2 b^2-8 a^2 b c-16 a^2 b n+8 a^2 c n+8 a^2 n^2-8 a b^2 c-16 a b^2 n+8 a b c^2+8 a b c d+48 a b c n-4 a b c+8 a b d^2+32 a b d n-8 a b d+64 a b n^2-16 a b n+2 a b-8 a c^2 d-24 a c^2 n+4 a c^2-16 a c d^2-72 a c d n+24 a c d-104 a c n^2+52 a c n-8 a c-8 a d^3-56 a d^2 n+20 a d^2-128 a d n^2+88 a d n-14 a d-112 a n^3+96 a n^2-30 a n+3 a+8 b^2 c n+8 b^2 n^2-8 b c^2 d-24 b c^2 n+4 b c^2-16 b c d^2-72 b c d n+24 b c d-104 b c n^2+52 b c n-8 b c-8 b d^3-56 b d^2 n+20 b d^2-128 b d n^2+88 b d n-14 b d-112 b n^3+96 b n^2-30 b n+3 b+8 c^3 d+16 c^3 n-4 c^3+16 c^2 d^2+80 c^2 d n-24 c^2 d+104 c^2 n^2-56 c^2 n+8 c^2+8 c d^3+80 c d^2 n-20 c d^2+232 c d n^2-128 c d n+14 c d+224 c n^3-180 c n^2+44 c n-3 c+16 d^3 n+104 d^2 n^2-40 d^2 n+224 d n^3-168 d n^2+28 d n+168 n^4-176 n^3+58 n^2-6 n$

When $(a,b,c,d) = (0,0,0,0)$, above equality reduces to $$\frac{\pi^2}{2} = \sum_{k\geq 0} \frac{4}{(1+2k)^2} = \sum_{n\geq 0} 64^{-n} \frac{(1/2)_n (1)_n^3}{(1/4)_n^2 (3/4)_n^2} \frac{3-23n+42n^2}{n^3(2n-1)}$$ which is exactly $(2)$. Those involving harmonic number $(9),(10),(11),(12)$ can be proved by comparing coefficient in Taylor expansion of above equality around $(a,b,c,d) = (0,0,0,0)$, coefficients of $\sum_{k\geq 0}$ will give the so-called multiple $t$-values (special case of alternating multiple zeta values), whose values, when weight is small, are well-understood.


For $(1)$ and $(4)$-$(7)$, one repeats the above process, except now using another $F(n,k)$: $F(n,k) = \frac{\Gamma (-a+c+2 n+1) \Gamma (a+k+n+1) \Gamma (-b+c+2 n+1) \Gamma (b+k+n+1)}{\Gamma (a-n) \Gamma (b-n) \Gamma (k+2 n+2) \Gamma (c+k+3 n+2) \Gamma (-a-b+c+3 n+1)}$

Unfortunately, one cannot deduce $(8)$ in this way.

For $(3)$ and $(13)$-$(15)$, one repeats the above process, with $F(n,k)$: $F(n,k) = \frac{\Gamma (-a+c+n+1) \Gamma (a+k+2 n+1) \Gamma (-b+c+n+1) \Gamma (b+k+2 n+1)}{\Gamma \left(a+\frac{1}{2}\right) \Gamma \left(b+\frac{1}{2}\right) \Gamma \left(k+2 n+\frac{3}{2}\right) \Gamma (c+k+3 n+2) \Gamma \left(-a-b+c+n+\frac{1}{2}\right)}$

$\endgroup$

You must log in to answer this question.

Start asking to get answers

Find the answer to your question by asking.

Ask question

Explore related questions

See similar questions with these tags.